Onco Practice Questions

You might also like

Download as docx, pdf, or txt
Download as docx, pdf, or txt
You are on page 1of 23

MS1 FINALS: Oncology Practice Questions

NCM 112 Care of Clients with Problems in Cellular Aberrations (Oncology)


Medical-Surgical Nursing 1
Finals
7. A female client is receiving chemotherapy to treat
1. A female client has an abnormal result on a breast cancer. Which assessment finding indicates a fluid
Papanicolaou test. After admitting, she read his chart and electrolyte imbalance induced by chemotherapy?
while the nurse was out of the room, the client asks what A. Urine output of 400 ml in 8 hours
dysplasia means. Which definition should the nurse B. Serum potassium level of 3.6 mEq/L
provide? C. Blood pressure of 120/64 to 130/72 mm Hg
A. Presence of completely undifferentiated tumor cells D. Dry oral mucous membranes and cracked lips
that don’t resemble cells of the tissues of their origin
B. Increase in the number of normal cells in a normal 8. Nurse April is teaching a group of women to perform
arrangement in a tissue or an organ breast self-examination. The nurse should explain that the
C. Replacement of one type of fully differentiated cell purpose of performing the examination is to discover:
by another in tissues where the second type A. cancerous lumps.
normally isn’t found B. areas of thickness or fullness.
D. Alteration in the size, shape, and organization of C. changes from previous self-examinations.
differentiated cells D. fibrocystic masses.

2. For a female client with newly diagnosed cancer, the 9. A client, age 41, visits the gynecologist. After examining
nurse formulates a nursing diagnosis of Anxiety related to her, the physician suspects cervical cancer. The nurse
the threat of death secondary to cancer diagnosis. Which reviews the client’s history for risk factors for this disease.
expected outcome would be appropriate for this client? Which history finding is a risk factor for cervical cancer?
A. “Client verbalizes feelings of anxiety.” A. Onset of sporadic sexual activity at age 17
B. “Client doesn’t guess at prognosis.” B. Spontaneous abortion at age 19
C. “Client uses any effective method to reduce tension.” C. Pregnancy complicated with eclampsia at age 27
D. “Client stops seeking information.” D. Human papillomavirus infection at age 32

3. A male client with a cerebellar brain tumor is admitted 10. A female client is receiving methotrexate (Mexate), 12
to an acute care facility. The nurse formulates a nursing g/m2 I.V., to treat osteogenic carcinoma. During
diagnosis of Risk for injury. Which “related-to” phrase methotrexate therapy, the nurse expects the client to
should the nurse add to complete the nursing diagnosis receive which other drug to protect normal cells?
statement? A. probenecid (Benemid)
A. Related to visual field deficits B. cytarabine (ara-C, cytosine arabinoside [Cytosar-U])
B. Related to difficulty swallowing C. thioguanine (6-thioguanine, 6-TG)
C. Related to impaired balance D. leucovorin (citrovorum factor or folinic acid
D. Related to psychomotor seizures [Wellcovorin])

4. A female client with cancer is scheduled for radiation 11. The nurse is interviewing a male client about his past
therapy. The nurse knows that radiation at any treatment medical history. Which preexisting condition may lead the
site may cause a certain adverse effect. Therefore, the nurse to suspect that a client has colorectal cancer?
nurse should prepare the client to expect: A. Duodenal ulcers
A. hair loss. B. Hemorrhoids
B. stomatitis. C. Weight gain
C. fatigue. D. Polyps
D. vomiting.
12. Nurse Amy is speaking to a group of women about
5. Nurse April is teaching a client who suspects that she early detection of breast cancer. The average age of the
has a lump in her breast. The nurse instructs the client women in the group is 47. Following the American Cancer
that a diagnosis of breast cancer is confirmed by: Society guidelines, the nurse should recommend that the
A. breast self-examination. women:
B. mammography. A. perform breast self-examination annually.
C. fine needle aspiration. B. have a mammogram annually.
D. chest X-ray. C. have a hormonal receptor assay annually.
D. have a physician conduct a clinical examination
every 2 years.
6. A male client undergoes a laryngectomy to treat
laryngeal cancer. When teaching the client how to care for 13. A male client with a nagging cough makes an
the neck stoma, the nurse should include which appointment to see the physician after reading that this
instruction? symptom is one of the seven warning signs of cancer.
A. “Keep the stoma uncovered.” What is another warning sign of cancer?
B. “Keep the stoma dry.” A. Persistent nausea
C. “Have a family member perform stoma care initially B. Rash
until you get used to the procedure.” C. Indigestion
D. “Keep the stoma moist.” D. Chronic ache or pain
14. For a female client newly diagnosed with radiation- 20. A female client is undergoing tests for multiple
induced thrombocytopenia, the nurse should include myeloma. Diagnostic study findings in multiple myeloma
which intervention in the plan of care? include:
A. Administering aspirin if the temperature exceeds A. a decreased serum creatinine level.
102° F (38.8° C) B. hypocalcemia.
B. Inspecting the skin for petechiae once every shift C. Bence Jones protein in the urine.
C. Providing for frequent rest periods D. a low serum protein level.
D. Placing the client in strict isolation
21. A 35 years old client has been receiving
15. Nurse Lucia is providing breast cancer education at a chemotherapy to treat cancer. Which assessment finding
community facility. The American Cancer Society suggests that the client has developed stomatitis
recommends that women get mammograms: (inflammation of the mouth)?
A. yearly after age 40. A. White, cottage cheese–like patches on the tongue
B. after the birth of the first child and every 2 years B. Yellow tooth discoloration
thereafter. C. Red, open sores on the oral mucosa
C. after the first menstrual period and annually D. Rust-colored sputum
thereafter.
D. every 3 years between ages 20 and 40 and annually 22. During chemotherapy, an oncology client has a
thereafter. nursing diagnosis of impaired oral mucous membrane
related to decreased nutrition and immunosuppression
16. Which intervention is appropriate for the nurse caring secondary to the cytotoxic effects of chemotherapy.
for a male client in severe pain receiving a continuous I.V. Which nursing intervention is most likely to decrease the
infusion of morphine? pain of stomatitis?
A. Assisting with a naloxone challenge test before A. Recommending that the client discontinue
therapy begins chemotherapy
B. Discontinuing the drug immediately if signs of B. Providing a solution of hydrogen peroxide and water
dependence appear for use as a mouth rinse
C. Changing the administration route to P.O. if the client C. Monitoring the client’s platelet and leukocyte counts
can tolerate fluids D. Checking regularly for signs and symptoms of
D. Obtaining baseline vital signs before administering stomatitis
the first dose
23. What should a male client over age 52 do to help
17. A 35 years old client with ovarian cancer is prescribed ensure early identification of prostate cancer?
hydroxyurea (Hydrea), an antimetabolite drug. A. Have a digital rectal examination and prostate-
Antimetabolites are a diverse group of antineoplastic specific antigen (PSA) test done yearly.
agents that interfere with various metabolic actions of the B. Have a transrectal ultrasound every 5 years.
cell. The mechanism of action of antimetabolites interferes C. Perform monthly testicular self-examinations,
with: especially after age 50.
A. cell division or mitosis during the M phase of the cell D. Have a complete blood count (CBC) and blood urea
cycle. nitrogen (BUN) and creatinine levels checked yearly.
B. normal cellular processes during the S phase of the
cell cycle. 24. A male client complains of sporadic epigastric pain,
C. the chemical structure of deoxyribonucleic acid yellow skin, nausea, vomiting, weight loss, and fatigue.
(DNA) and chemical binding between DNA Suspecting gallbladder disease, the physician orders a
molecules (cell cycle–nonspecific). diagnostic workup, which reveals gallbladder cancer.
D. one or more stages of ribonucleic acid (RNA) Which nursing diagnosis may be appropriate for this
synthesis, DNA synthesis, or both (cell cycle– client?
nonspecific). A. Anticipatory grieving
B. Impaired swallowing
18. The ABCD method offers one way to assess skin C. Disturbed body image
lesions for possible skin cancer. What does the A stand D. Chronic low self-esteem
for?
A. Actinic 25. A male client is in isolation after receiving an internal
B. Asymmetry radioactive implant to treat cancer. Two hours later, the
C. Arcus nurse discovers the implant in the bed linens. What
D. Assessment should the nurse do first?
A. Stand as far away from the implant as possible and
19. When caring for a male client diagnosed with a brain call for help.
tumor of the parietal lobe, the nurse expects to assess: B. Pick up the implant with long-handled forceps and
A. short-term memory impairment. place it in a lead-lined container.
B. tactile agnosia. C. Leave the room and notify the radiation therapy
C. seizures. department immediately.
D. contralateral homonymous hemianopia. D. Put the implant back in place, using forceps and a
shield for self-protection, and call for help.
26. Jeovina, with advanced breast cancer is prescribed D. immediately after her menstrual period.
tamoxifen (Nolvadex). When teaching the client about this
drug, the nurse should emphasize the importance of 3. Nurse Kent is teaching a male client to perform monthly
reporting which adverse reaction immediately? testicular self-examinations. Which of the following points
A. Vision changes would be appropriate to make?
B. Hearing loss A. Testicular cancer is a highly curable type of cancer.
C. Headache B. Testicular cancer is very difficult to diagnose.
D. Anorexia C. Testicular cancer is the number one cause of cancer
deaths in males.
27. A female client with cancer is being evaluated for D. Testicular cancer is more common in older men.
possible metastasis. Which of the following is one of the
most common metastasis sites for cancer cells? 4. Rhea, has malignant lymphoma. As part of her
A. Liver chemotherapy, the physician prescribes chlorambucil
B. Colon (Leukeran), 10 mg by mouth daily. When caring for the
C. Reproductive tract client, the nurse teaches her about adverse reactions to
D. White blood cells (WBCs) chlorambucil, such as alopecia. How soon after the first
administration of chlorambucil might this reaction occur?
28. A 34-year-old female client is requesting information A. Immediately
about mammograms and breast cancer. She isn’t B. 1 week
considered at high risk for breast cancer. What should the C. 2 to 3 weeks
nurse tell this client? D. 1 month
A. She should have had a baseline mammogram before
age 30. 5. A male client is receiving the cell cycle–nonspecific
B. She should eat a low-fat diet to further decrease her alkylating agent thiotepa (Thioplex), 60 mg weekly for 4
risk of breast cancer. weeks by bladder instillation as part of a
C. She should perform breast self-examination during chemotherapeutic regimen to treat bladder cancer. The
the first 5 days of each menstrual cycle. client asks the nurse how the drug works. How does
D. When she begins having yearly mammograms, thiotepa exert its therapeutic effects?
breast self-examinations will no longer be necessary. A. It interferes with deoxyribonucleic acid (DNA)
replication only.
29. Nurse Brian is developing a plan of care for marrow B. It interferes with ribonucleic acid (RNA) transcription
suppression, the major dose-limiting adverse reaction to only.
floxuridine (FUDR). How long after drug administration C. It interferes with DNA replication and RNA
does bone marrow suppression become noticeable? transcription.
A. 24 hours D. It destroys the cell membrane, causing lysis.
B. 2 to 4 days
C. 7 to 14 days 6. The nurse is instructing the 35 year old client to perform
D. 21 to 28 days a testicular self-examination. The nurse tells the client:
30. The nurse is preparing for a female client for magnetic A. To examine the testicles while lying down
resonance imaging (MRI) to confirm or rule out a spinal B. That the best time for the examination is after a
cord lesion. During the MRI scan, which of the following shower
would pose a threat to the client? C. To gently feel the testicle with one finger to feel for a
A. The client lies still. growth
B. The client asks questions. D. That testicular self-examination should be done at
C. The client hears thumping sounds. least every 6 months
D. The client wears a watch and wedding band.
7. A female client with cancer is receiving chemotherapy
and develops thrombocytopenia. The nurse identifies
1. Nina, an oncology nurse educator is speaking to a
which intervention as the highest priority in the nursing
women’s group about breast cancer. Questions and
plan of care?
comments from the audience reveal a misunderstanding
A. Monitoring temperature
of some aspects of the disease. Various members of the
B. Ambulation three times daily
audience have made all of the following statements.
C. Monitoring the platelet count
Which one is accurate?
D. Monitoring for pathological fractures
A. Mammography is the most reliable method for
detecting breast cancer.
8. Gian, a community health nurse is instructing a group
B. Breast cancer is the leading killer of women of
of female clients about breast self-examination. The nurse
childbearing age.
instructs the client to perform the examination:
C. Breast cancer requires a mastectomy.
A. At the onset of menstruation
D. Men can develop breast cancer.
B. Every month during ovulation
C. Weekly at the same time of day
2. Nurse Meredith is instructing a premenopausal woman
D. 1 week after menstruation begins
about breast self-examination. The nurse should tell the
client to do her self-examination:
A. at the end of her menstrual cycle.
B. on the same day each month.
C. on the 1st day of the menstrual cycle.
16. Nurse Joy is caring for a client with an internal
9. Nurse Cecilia is caring for a client who has undergone radiation implant. When caring for the client, the nurse
a vaginal hysterectomy. The nurse avoids which of the should observe which of the following principles?
following in the care of this client? A. Limit the time with the client to 1 hour per shift
A. Elevating the knee gatch on the bed B. Do not allow pregnant women into the client’s room
B. Assisting with range-of-motion leg exercises C. Remove the dosimeter badge when entering the
C. Removal of antiembolism stockings twice daily client’s room
D. Checking placement of pneumatic compression D. Individuals younger than 16 years old may be
boots allowed to go in the room as long as they are 6 feet
away from the client
10. Mina, who is suspected of an ovarian tumor is
scheduled for a pelvic ultrasound. The nurse provides 17. A cervical radiation implant is placed in the client for
which preprocedure instruction to the client? treatment of cervical cancer. The nurse initiates what
A. Eat a light breakfast only most appropriate activity order for this client?
B. Maintain an NPO status before the procedure A. Bed rest
C. Wear comfortable clothing and shoes for the B. Out of bed ad lib
procedure C. Out of bed in a chair only
D. Drink six to eight glasses of water without voiding D. Ambulation to the bathroom only
before the test
18. A female client is hospitalized for insertion of an
11. A male client is diagnosed as having a bowel tumor internal cervical radiation implant. While giving care, the
and several diagnostic tests are prescribed. The nurse nurse finds the radiation implant in the bed. The initial
understands that which test will confirm the diagnosis of action by the nurse is to:
malignancy? A. Call the physician
A. Biopsy of the tumor B. Reinsert the implant into the vagina immediately
B. Abdominal ultrasound C. Pick up the implant with gloved hands and flush it
C. Magnetic resonance imaging down the toilet
D. Computerized tomography scan D. Pick up the implant with long-handled forceps and
place it in a lead container.
12. A female client diagnosed with multiple myeloma and
the client asks the nurse about the diagnosis. The nurse 19. The nurse is caring for a female client experiencing
bases the response on which description of this disorder? neutropenia as a result of chemotherapy and develops a
A. Altered red blood cell production plan of care for the client. The nurse plans to:
B. Altered production of lymph nodes A. Restrict all visitors
C. Malignant exacerbation in the number of leukocytes B. Restrict fluid intake
D. Malignant proliferation of plasma cells within the C. Teach the client and family about the need for hand
bone hygiene
D. Insert an indwelling urinary catheter to prevent skin
13. Nurse Bea is reviewing the laboratory results of a breakdown
client diagnosed with multiple myeloma. Which of the
following would the nurse expect to note specifically in this 20. The home health care nurse is caring for a male client
disorder? with cancer and the client is complaining of acute pain.
A. Increased calcium The appropriate nursing assessment of the client’s pain
B. Increased white blood cells would include which of the following?
C. Decreased blood urea nitrogen level A. The client’s pain rating
D. Decreased number of plasma cells in the bone B. Nonverbal cues from the client
marrow C. The nurse’s impression of the client’s pain
D. Pain relief after appropriate nursing intervention
14. Vanessa, a community health nurse conducts a health
promotion program regarding testicular cancer to 21. Nurse Mickey is caring for a client who is
community members. The nurse determines that further postoperative following a pelvic exenteration and the
information needs to be provided if a community member physician changes the client’s diet from NPO status to
states that which of the following is a sign of testicular clear liquids. The nurse makes which priority assessment
cancer? before administering the diet?
A. Alopecia A. Bowel sounds
B. Back pain B. Ability to ambulate
C. Painless testicular swelling C. Incision appearance
D. Heavy sensation in the scrotum D. Urine specific gravity

15. The male client is receiving external radiation to the 22. A male client is admitted to the hospital with a
neck for cancer of the larynx. The most likely side effect to suspected diagnosis of Hodgkin’s disease. Which
be expected is: assessment findings would the nurse expect to note
A. Dyspnea specifically in the client?
B. Diarrhea A. Fatigue
C. Sore throat B. Weakness
D. Constipation C. Weight gain
D. Enlarged lymph nodes 30. The client with leukemia is receiving busulfan
23. During the admission assessment of a 35 year old (Myleran) and allopurinol (Zyloprim). The nurse tells the
client with advanced ovarian cancer, the nurse recognizes client that the purpose if the allopurinol is to prevent:
which symptom as typical of the disease? A. Nausea
A. Diarrhea B. Alopecia
B. Hypermenorrhea C. Vomiting
C. Abdominal bleeding D. Hyperuricemia
D. Abdominal distention
31. A 25 year old patient is inquiring about the methods or
24. Nurse Kate is reviewing the complications of ways to detect cancer earlier. The nurse least likely
colonization with a client who has microinvasive cervical identify this method by stating:
cancer. Which complication, if identified by the client, A. Annual chest x-ray.
indicates a need for further teaching? B. Annual Pap smear for sexually active women only.
A. Infection C. Annual digital rectal examination for persons over
B. Hemorrhage age 40.
C. Cervical stenosis D. Yearly physical and blood examination
D. Ovarian perforation
32. The removal of entire breast, pectoralis major and
25. Mr. Miller has been diagnosed with bone cancer. You minor muscles and neck lymph nodes which is followed
know this type of cancer is classified as: by skin grafting is a procedure called:
A. sarcoma. A. Simple mastectomy
B. lymphoma. B. Modified radical mastectomy
C. carcinoma. C. Radiation therapy
D. melanoma. D. Radical mastectomy

26. Sarah, a hospice nurse visits a client dying of ovarian 33. Chemotherapy is one of the therapeutic modalities for
cancer. During the visit, the client expresses that “If I can cancer. This treatment is contraindicated to which of the
just live long enough to attend my daughter’s graduation, following conditions?
I’ll be ready to die.” Which phrase of coping is this client A. Recent surgery
experiencing? B. Pregnancy
A. Anger C. Bone marrow depression
B. Denial D. All of the above
C. Bargaining
D. Depression 34. The nurse is preparing Cyclophosphamide (Cytoxan).
Safe handling of the drug should be implemented to
27. Nurse Farah is caring for a client following a protect the nurse from injury. Which of the following action
mastectomy. Which assessment finding indicates that the by the nurse should be corrected?
client is experiencing a complication related to the A. The nurse should wear mask and gloves.
surgery? B. Air bubbles should be expelled on wet cotton.
A. Pain at the incisional site C. Label the hanging IV bottle with “ANTINEOPLASTIC
B. Arm edema on the operative side CHEMOTHERAPY” sign.
C. Sanguineous drainage in the Jackson-Pratt drain D. Vent vials after mixing.
D. Complaints of decreased sensation near the
operative site 35. Neoplasm can be classified as either benign or
malignant. The following are characteristics of malignant
28. The nurse is admitting a male client with laryngeal tumor apart from:
cancer to the nursing unit. The nurse assesses for which A. Metastasis
most common risk factor for this type of cancer? B. Infiltrates surrounding tissues
A. Alcohol abuse C. Encapsulated
B. Cigarette smoking D. Poorly differentiated cells
C. Use of chewing tobacco
D. Exposure to air pollutants 36. On a clinic visit a client who has a relative with cancer,
is asking about the warning signs that may relate to
29. The female client who has been receiving radiation cancer. The nurse correctly identifies the warning signs of
therapy for bladder cancer tells the nurse that it feels as if cancer by responding:
she is voiding through the vagina. The nurse interprets A. “If a sore healing took a month or more to heal,
that the client may be experiencing: cancer should be suspected.”
A. Rupture of the bladder B. “Presence of dry cough is one of the warning signs
B. The development of a vesicovaginal fistula of cancer.”
C. Extreme stress caused by the diagnosis of cancer C. “A lump located only in the breast area may suggest
D. Altered perineal sensation as a side effect of the presence of cancer.”
radiation therapy D. “Sudden weight loss of unexplained etiology can be
a warning sign of cancer.”
37. In staging and grading neoplasm TNM system is used. 44. The following are teaching guidelines regarding
TNM stands for: radiation therapy except:
A. Time, neoplasm, mode of growth A. The therapy is painless
B. Tumor, node, metastasis B. To promote safety, the client is assisted by therapy
C. Tumor, neoplasm, mode of growth personnel while the machine is in operation.
D. Time, node, metastasis C. The client may communicate all his concerns or
needs or discomforts while the machine is operating.
38. Breast self examination (BSE) is one of the ways to D. Safety precautions are necessary only during the
detect breast cancer earlier. The nurse is conducting a time of actual irradiation.
health teaching to female clients in a clinic. During
evaluation the clients are asked to state what they 45. Contact of client on radiation therapy should be limited
learned. Which of the following statement made by a only to how many minutes to promote safety of the
client needs further teaching about BSE? therapy personnel?
A. “BSE is done after menstruation.” A. 1 minute
B. “BSE palpation is done by starting at the center B. 3 minutes
going to the periphery in a circular motion.” C. 5 minutes
C. “BSE can be done in either supine or standing D. 10 minutes
position.”
D. “BSE should start from age 20.” 46. A client is taking Cyclophosphamide (Cytoxan) for the
treatment of lymphoma. The nurse is very cautious in
39. A client had undergone radiation therapy (external). administering the medication because this drug poses the
The expected side effects include the following apart from: fatal side effect of:
A. Hair loss A. Alopecia
B. Ulceration of oral mucous membranes B. Myeloma
C. Constipation C. CNS toxicity
D. Headache D. Hemorrhagic cystitis

40. Nurse Janet is assigned in the oncology section of the 47. Cytarabine (Ara-C) is an antimetabolite that can cause
hospital. Which of the following orders should the nurse a common cytarabine syndrome which includes the
question if a client is on radiation therapy? following apart from:
A. Analgesics before meals A. Fever
B. Saline rinses every 2 hours B. Myalgia
C. Aspirin every 4 hours C. Chest pain
D. Bland diet D. Diarrhea

41. Skin reactions are common in radiation therapy. 48. To provide relief from the cytarabine syndrome, which
Nursing responsibilities on promoting skin integrity should drug is given?
be promoted apart from: A. Analgesic
A. Avoiding the use of ointments, powders and lotion to B. Aspirin
the area C. Steroids
B. Using soft cotton fabrics for clothing D. Allopurinol
C. Washing the area with a mild soap and water and
patting it dry not rubbing it. 49. Chemotherapeutic agents have different specific
D. Avoiding direct sunshine or cold. classifications. The following medications are
antineoplastic antibiotics except:
42. Nausea and vomiting is an expected side effect of A. Doxorubicin (Adriamycin)
chemotherapeutic drug use. Which of the following drug B. Fluorouracil (Adrucil)
should be administered to a client on chemotherapy to C. Mitoxantrone (Novantrone)
prevent nausea and vomiting? D. Bleomycin (Blenoxane)
A. Metochlopramide (Metozol)
B. Succimer (Chemet) 50. Specific classification of the chemotherapeutic agent,
C. Anastrazole (Arimidex) Vincristine (Oncovin) is:
D. Busulfan (Myleran) A. Hormone modulator
B. Mitotic inhibitor
43. Radiation protection is very important to implement C. Antineoplastic antibiotic
when performing nursing procedures. When the nurse is D. Antimetabolite
not performing any nursing procedures what distance
should be maintained from the client? 51. A client is diagnosed with progressive prostate cancer.
A. 1 feet The nurse expects which drug is given?
B. 2 feet A. Anstrazole (arimidex)
C. 2.5 feet B. Estramustine (Emcyt)
D. 3 feet C. Paclitaxel (Taxol)
D. Irinotecan (Camptosar)
52. A client taking a chemotherapeutic agent understands 5. Which of the following statements is correct about
the effects of therapy by stating: the rate of cell growth in relation to chemotherapy?
A. “I will avoid eating hot and spicy foods.” A. Faster growing cells are less susceptible to
B. “I should stay in my room all the time.” chemotherapy.
C. “I should limit my fluid intake to about 500 ml per B. Non-dividing cells are more susceptible to
day.” chemotherapy
D. “I should notify the physician immediately if a urine C. Faster growing cells are more susceptible to
color change is observed.” chemotherapy
D. Slower growing cells are more susceptible to
53. A client is diagnosed with breast cancer. The tumor chemotherapy.
size is up to 5 cm with axillary and neck lymph node
involvement. The client is in what stage of breast cancer? 6. Which of the following foods should a client with
A. Stage I leukemia avoid?
B. Stage II A. White bread
C. Stage III B. Carrot sticks
D. Stage IV C. Stewed apples
D. Medium rare steak
54. The classic symptoms that define breast cancer
includes the following except: 7. A client with leukemia has neutropenia. Which of the
A. “pink peel” skin following functions must be frequently assessed?
B. Solitary, irregularly shaped mass A. Blood pressure
C. Firm, nontender, nonmobile mass B. Bowel sounds
D. Abnormal discharge from the nipple C. Heart sounds
D. Breath sounds
55. Surgical procedure to treat breast cancer involves the
removal of the entire breast, pectoralis major muscle and 8. Which of the following clients is most at risk for
the axillary lymph nodes is: developing multiple myeloma?
A. Simple mastectomy A. A 20-year-old Asian woman
B. Modified radical mastectomy B. A 30-year-old White man
C. Halstead Surgery C. A 50-year-old Hispanic woman
D. Radical mastectomy D. A 60-year-old Black man

1. Which of the following conditions is not a complication 9. Which of the following substances has abnormal
of Hodgkin’s disease? values early in the course of multiple myeloma (MM)?
A. Anemia A. Immunoglobulins
B. Infection B. Platelets
C. Myocardial Infarction C. Red blood cells
D. Nausea D. White blood cells

2. Which of the following laboratory values is expected 10. For which of the following conditions is a client with
for a client just diagnosed with chronic lymphocytic multiple myeloma (MM) monitored?
leukemia? A. Hypercalcemia
A. Elevated sedimentation rate B. Hyperkalemia
B. Uncontrolled proliferation of granulocytes C. Hypernatremia
C. Thrombocytopenia and increased lymphocytes D. Hypermagnesemia
D. Elevated aspartate aminotransferase and alanine
aminotransferase levels. 11. Giving instructions for breast self-examination is
particularly important for clients with which of the following
3. At the time of diagnosis of Hodgkin’s lymphoma, medical problems?
which of the following areas is often involved? A. Cervical dysplasia
A. Back B. A dermoid cyst
B. Chest C. Endometrial polyps
C. Groin D. Ovarian cancer
D. Neck
12. During a routine physical examination, a firm mass is
4. According to a standard staging classification of palpated in the right breast of a 35-year-old woman.
Hodgkin’s disease, which of the following criteria reflects Which of the following findings or client history would
stage II? suggest cancer of the breast as opposed to fibrocystic
A. Involvement of extralymphatic organs or tissues disease?
B. Involvement of single lymph node region or structure A. History of early menarche
C. Involvement of two or more lymph node regions or B. Cyclic changes in mass size
structures. C. History of anovulatory cycles
D. Involvement of lymph node regions or structures on D. Increased vascularity of the breast
both sides of the diaphragm.
13. The client with which of the following types of lung 21. The client with a benign lung tumor is treated in which
cancer has the best prognosis? of the following ways?
A. Adenocarcinoma A. The tumor is treated with radiation only.
B. Oat cell B. The tumor is treated with chemotherapy only.
C. Squamous cell C. The tumor is left alone unless symptoms are present.
D. Small cell D. The tumor is removed, involving the least possible
amount of tissue.
14. Warning signs and symptoms of lung cancer include
persistent cough, bloody sputum, dyspnea, and which of 22. In the client with terminal lung cancer, the focus of
the other following symptoms? nursing care is on which of the following nursing
A. Dizziness interventions?
B. Generalized weakness A. Provide emotional support
C. Hypotension B. Provide nutritional support
D. Recurrent pleural effusion C. Provide pain control
D. Prepare the client’s will
15. A centrally located tumor would produce which of the
following symptoms? 23. What are the three most important prognostic factors
A. Coughing in determining long-term survival for children with acute
B. Hemoptysis leukemia?
C. Pleuritic pain A. Histologic type of disease, initial platelet count, and
D. Shoulder pain type of treatment
B. Type of treatment and client’s sex
16. Which of the following interventions is the key to C. Histologic type of disease, initial WBC count, and
increasing the survival rates of clients with lung cancer? client’s age at diagnosis
A. Early bronchoscopy D. Progression of illness, WBC at the time of diagnosis,
B. Early detection and client’s age at the time of diagnosis.
C. High-dose chemotherapy
D. Smoking cessation 24. Which of the following complications are three main
consequences of leukemia?
17. A client has been diagnosed with lung cancer and A. Bone deformities, spherocytosis, and infection.
requires a wedge resection. How much of the lung is B. Anemia, infection, and bleeding tendencies
removed? C. Lymphocytopoesis, growth delays, and hirsutism
A. One entire lung D. Polycythemia, decreased clotting time, and infection.
B. A lobe of the lung
C. A small, localized area near the surface of the lung. 25. A child is seen in the pediatrician’s office for
D. A segment of the lung, including a bronchiole and its complaints of bone and joint pain. Which of the following
alveoli. other assessment findings may suggest leukemia?
A. Abdominal pain
18. When a client has a lobectomy, what fills the space B. Increased activity level
where the lobe was? C. Increased appetite
A. The space stays empty. D. Petechiae
B. The surgeon fills the space with gel
C. The lung space fills up with serous fluid 26. Which of the following assessment findings in a client
D. The remaining lobe or lobes overexpand to fill the with leukemia would indicate that the cancer has invaded
space. the brain?
A. Headache and vomiting.
19. Which of the following is the primary goal for surgical B. Restlessness and tachycardia
resection of lung cancer? C. Hypervigilant and anxious behavior
A. To remove the tumor and all surrounding tissue. D. Increased heart rate and decreased blood pressure.
B. To remove the tumor and as little surrounding tissue
as possible. 27. Which of the following types of leukemia carries the
C. To remove all of the tumor and any collapsed alveoli best prognosis?
in the same region. A. Acute lymphoblastic leukemia
D. To remove as much as the tumor as possible, B. Acute myelogenous leukemia
without removing any alveoli. C. Basophillic leukemia
D. Eosinophillic leukemia
20. If the client with lung cancer also has preexisting
pulmonary disease, which of the following statements best 28. Which of the following is the reason to perform a
describes how the extent of that can be performed? spinal tap on a client newly diagnosed with leukemia?
A. It doesn’t affect it. A. To rule out meningitis
B. It may require a whole lung to be removed. B. To decrease intracranial pressure
C. The entire tumor may not be able to be removed C. To aid in classification of the leukemia
D. It may prevent surgery if the client can’t tolerate lung D. To assess for central nervous system infiltration
tissue removal.
29. Which of the following tests in performed on a client 37. The community nurse is conducting a health
with leukemia before initiation of therapy to evaluate the promotion program at a local school and is discussing the
child’s ability to metabolize chemotherapeutic agents? risk factors associated with cancer. Which of the following,
A. Lumbar puncture if identified by the client as a risk factor, indicates a need
B. Liver function studies for further instructions?
C. Complete blood count (CBC) A. Viral factors
D. Peripheral blood smear B. Stress
C. Low-fat and high-fiber diets
30. Which of the following immunizations should not be D. Exposure to radiation
given to a 4-month-old sibling of a client with leukemia?
A. Diphtheria and tetanus and pertussis (DPT) vaccine. 38. The client with cancer is receiving chemotherapy and
B. Hepatitis B vaccine develops thrombocytopenia. The nurse identifies which
C. Haemophilus influenza type b vaccines (Hib) intervention as the highest priority in the nursing plan of
D. Oral poliovirus vaccine (OPV) care?
A. Ambulation three times a day
31. Which of the following medications usually is given to B. Monitoring temperature
a client with leukemia as prophylaxis against P. C. Monitoring the platelet count
carinii pneumonia? D. Monitoring for pathological factors
A. Bactrim
B. Oral nystatin suspension 39. A client is diagnosed with multiple myeloma. The
C. Prednisone client asks the nurse about the diagnosis. The nurse
D. Vincristine (Oncovin) bases the response on which of the following descriptions
of this disorder?
32. In which of the following diseases would bone marrow A. Malignant exacerbation in the number of leukocytes.
transplantation not be indicated in a newly diagnosed B. Altered red blood cell production.
client? C. Altered production of lymph nodes
A. Acute lymphocytic leukemia D. Malignant proliferation of plasma cells and tumors
B. Chronic myeloid leukemia within the bone.
C. Severe aplastic anemia
D. Severe combined immunodeficiency 40. The nurse is reviewing the laboratory results of a
client diagnosed with multiple myeloma. Which of the
33. Which of the following treatment measures should be following would the nurse expect to note specifically in this
implemented for a child with leukemia who has been disorder?
exposed to the chickenpox? A. Decreased number of plasma cells in the bone
A. No treatment is indicated. marrow.
B. Acyclovir (Zovirax) should be started on exposure B. Increased WBC’s
C. Varicella-zoster immune globin (VZIG) should be C. Increased calcium levels
given with the evidence of disease D. Decreased blood urea nitrogen
D. VZIG should be given within 72 hours of exposure.
41. The nurse is developing a plan of care for the client
34. Nausea and vomiting are common adverse effects of with multiple myeloma. The nurse includes which priority
radiation and chemotherapy. When should a nurse intervention in the plan of care?
administer antiemetics? A. Coughing and deep breathing
A. 30 minutes before the initiation of therapy. B. Encouraging fluids
B. With the administration of therapy. C. Monitoring red blood cell count
C. Immediately after nausea begins. D. Providing frequent oral care
D. When therapy is completed.
42. The oncology nurse specialist provides an
35. Parents of pediatric clients who undergo irradiation educational session to nursing staff regarding the
involving the central nervous system should be warned characteristics of Hodgkin’s disease. The nurse
about postirradiation somnolence. When does this determines that further education is needed if a nursing
neurologic syndrome usually occur? staff member states that which of the following is
A. Immediately characteristic of the disease?
B. Within 1 to 2 weeks A. Presence of Reed-Sternberg cells
C. Within 5 to 8 weeks B. Involvement of lymph nodes, spleen, and liver
D. Within 3 to 6 months C. Occurs most often in the older client
D. Prognosis depends on the stage of the disease
36. The nurse is instructing the client to perform a
testicular self-examination. The nurse tells the client: 43. The nurse is reviewing the laboratory results of a
A. To examine the testicles while lying down. client receiving chemotherapy. The platelet count is
B. The best time for the examination is after a shower 10,000 cells/mm. Based on this laboratory value, the
C. To gently feel the testicle with one finger to feel for a priority nursing assessment is which of the following?
growth A. Assess level of consciousness
D. That testicular examination should be done at least B. Assess temperature
every 6 months. C. Assess bowel sounds
D. Assess skin turgor
44. The nurse is caring for a client following a modified 50. A client with stomach cancer is admitted to the
radical mastectomy. Which assessment finding would oncology unit after vomiting for 3 days. Physical
indicate that the client is experiencing a complication assessment findings include irregular pulse, muscle
related to this surgery? twitching, and complaints of prickling sensations in the
A. Sanguineous drainage in the Jackson-Pratt drain fingers and hands. Laboratory results include a potassium
B. Pain at the incisional site level of 2.9 mEq/L, a pH of 7.46, and a bicarbonate level
C. Complaints of decreased sensation near the of 29 mEq/L. The client is experiencing:
operative site A. Respiratory alkalosis
D. Arm edema on the operative side B. Respiratory acidosis
C. Metabolic alkalosis
45. A nurse is providing education in a community setting D. Metabolic acidosis
about general measures to avoid excessive sun exposure.
Which of the following recommendations is appropriate? 51. A 32-year-old woman meets with the nurse on her
A. Apply sunscreen only after going in the water. first office visit since undergoing a left mastectomy. When
B. Avoid peak exposure hours from 9am to 1pm asked how she is doing, the woman states her appetite is
C. Wear loosely woven clothing for added ventilation still not good, she is not getting much sleep because she
D. Apply sunscreen with a sun protection factor (SPF) doesn’t go to bed until her husband is asleep, and she is
of 15 or more before sun exposure. really anxious to get back to work. Which of the following
nursing interventions should the nurse explore to support
46. Which of the following nursing interventions would be the client’s current needs?
most helpful in making the respiratory effort of a client with A. Call the physician to discuss allowing the client to
metastatic lung cancer more efficient? return to work earlier.
A. Teaching the client diaphragmatic breathing B. Suggest that the client learn relaxation techniques to
techniques help with her insomnia
B. Administering cough suppressants as ordered C. Perform a nutritional assessment to assess for
C. Teaching and encouraging pursed-lip breathing anorexia
D. Placing the client in a low semi-Fowlers position D. Ask open-ended questions about sexuality issues
related to her mastectomy
47. The nurse is teaching a 17-year old client and the
client’s family about what to expect with high-dose 52. One of the most serious blood coagulation
chemotherapy and the effects of neutropenia. What complications for individuals with cancer and for those
should the nurse teach as the most reliable early indicator undergoing cancer treatments is disseminated
of infection in a neutropenic client? intravascular coagulation (DIC). The most common cause
A. Fever of this bleeding disorder is:
B. Chills A. Underlying liver disease
C. Tachycardia B. Brain metastasis
D. Dyspnea C. Intravenous heparin therapy
D. Sepsis
48. A 58-year-old man is going to have chemotherapy for
lung cancer. He asks the nurse how the chemotherapeutic 53. A pneumonectomy is a surgical procedure sometimes
drugs will work. The most accurate explanation the nurse indicated for treatment of non-small-cell lung cancer. A
can give is which of the following? pneumonectomy involves removal of:
A. “Chemotherapy affects all rapidly dividing cells.” A. An entire lung field
B. “The molecular structure of the DNA is altered.” B. A small, wedge-shaped lung surface
C. “Cancer cells are susceptible to drug toxins.” C. One lobe of a lung
D. “Chemotherapy encourages cancer cells to divide.” D. One or more segments of a lung lobe

49. When caring for a client with a central venous line, 54. A 36-year-old man with lymphoma presents with
which of the following nursing actions should be signs of impending septic shock 9 days after
implemented in the plan of care for chemotherapy chemotherapy. The nurse could expect which of the
administration? Select all that apply. following to be present?
A. Verify patency of the line by the presence of a blood A. Flushing, decreased oxygen saturation, mild
return at regular intervals. hypotension
B. Inspect the insertion site for swelling, erythema, or B. Low-grade fever, chills, tachycardia
drainage. C. Elevated temperature, oliguria, hypotension
C. Administer a cytotoxic agent to keep the regimen on D. High-grade fever, normal blood pressure, increased
schedule even if blood return is not present. respirations
D. If unable to aspirate blood, reposition the client and
encourage the client to cough. 55. Which of the following represents the most
E. Contact the health care provider about verifying appropriate nursing intervention for a client with pruritis
placement if the status is questionable. caused by cancer or the treatments?
A. Administration of antihistamines
B. Steroids
C. Silk sheets
D. Medicated cool baths
56. A 56-year-old woman is currently receiving radiation D. A patient who needs central line insertion for
therapy to the chest wall for recurrent breast cancer. She chemotherapy
calls her health care provider to report that she has pain 6. A community health center is preparing a presentation
while swallowing and burning and tightness in her chest. on the prevention and detection of cancer. Which health
Which of the following complications of radiation therapy care professional (RN, LPN/LVN, nurse practitioner,
is most likely responsible for her symptoms? nutritionist) should be assigned to address the following
A. Hiatal hernia topics?
B. Stomatitis A. Explain screening exams and diagnostic testing for
C. Radiation enteritis common cancers
D. Esophagitis ____________________________
B. How to plan a balanced diet and reduce fats and
MSN Exams preservatives _______________________
C. Prepare a poster on the seven warning signs of
1. You are caring for a patient with esophageal cancer. cancer ____________________________
Which task could be delegated to the nursing assistant? D. How to practice breast or testicular self-examination
A. Assist the patient with oral hygiene. _______________________
B. Observe the patient’s response to feedings. E. Strategies for reducing risk factors such as smoking
C. Facilitate expression of grief or anxiety. and obesity
D. Initiate daily weights. ___________________________

2. A 56-year-old patient comes to the walk-in clinic for 7. The physician tells the patient that there will be an initial
scant rectal bleeding and intermittent diarrhea and course of treatment with continued maintenance
constipation for the past several months. There is a treatments and ongoing observation for signs and
history of polyps and a family history for colorectal cancer. symptoms over a prolonged period of time. You can help
While you are trying to teach about colonoscopy, the the patient by reinforcing that the primary goal for this type
patient becomes angry and threatens to leave. What is of treatment is:
the priority diagnosis? A. Cure
A. Diarrhea/Constipation related to altered bowel B. Control
patterns C. Palliation
B. Knowledge Deficit related to disease process and D. Permanent remission
diagnostic procedure
C. Risk for Fluid Volume Deficit related to rectal 8. For a patient who is experiencing side effects of
bleeding and diarrhea radiation therapy, which task would be the most
D. Anxiety related to unknown outcomes and perceive appropriate to delegate to the nursing assistant?
threat to body integrity A. Assist the patient to identify patterns of fatigue.
B. Recommend participation in a walking program.
3. Which patient is at greatest risk for pancreatic cancer? C. Report the amount and type of food consumed from
A. An elderly black male with a history of smoking and the tray.
alcohol use D. Check the skin for redness and irritation after the
B. A young, white obese female with no known health treatment.
issues
C. A young black male with juvenile onset diabetes 9. For a patient on the chemotherapeutic drug vincristine
D. An elderly white female with a history of pancreatitis (Oncovin), which of the following side effects should be
reported to the physician?
4. The disease progress of cancers, such as cervical or A. Fatigue
Hodgkin’s, can be classified according to a clinical staging B. Nausea and vomiting
system. Place the description of stages 0-IV in the correct C. Paresthesia
order. D. Anorexia
A. Metastasis
B. Limited local spread 10.For a patient who is receiving chemotherapy, which
C. Cancer in situ laboratory result is of particular importance?
D. Tumor limited to tissue of origin A. WBC
E. Extensive local and regional spread B. PT and PTT
___C__, __B___, __D___, ___E__, __A__ C. Electrolytes
D. BUN
5. In assigning patients with alterations related to
gastrointestinal (GI) cancer, which would be the most 11.For care of a patient who has oral cancer, which task
appropriate nursing care tasks to assign to the LPN/LVN, would be appropriate to delegate to the LPN/LVN?
under supervision of the team leader RN? A. Assist the patient to brush and floss.
A. A patient with severe anemia secondary to GI B. Explain when brushing and flossing are
bleeding contraindicated.
B. A patient who needs enemas and antibiotics to C. Give antacids and sucralfate suspension as ordered.
control GI bacteria D. Recommend saliva substitutes.
C. A patient who needs pre-op teaching for bowel
resection surgery
12.When assigning staff to patients who are receiving 19.What do you tell patients is the most important risk
chemotherapy, what is the major consideration about factor for lung cancer when you are teaching about lung
chemotherapeutic drugs? cancer prevention?
A. During preparation, drugs may be absorbed through A. Cigarette smoking
the skin or inhaled. B. Exposure to environmental/occupational carcinogens
B. Many chemotherapeutics are vesicants. C. Exposure to environmental tobacco smoke (ETS)
C. Chemotherapeutics are frequently given through D. Pipe or cigar smoking
central nervous access devices.
D. Oral and venous routes are the most common. 20.Following chemotherapy, a patient is being closely
monitored for tumor lysis syndrome. Which laboratory
13.You have just received the morning report from the value requires particular attention?
night shift nurses. List the order of priority for assessing A. Platelet count
and caring for these patients. B. Electrolytes
A. A patient who developed tumor lysis syndrome C. Hemoglobin
around 5:00 AM D. Hematocrit
B. A patient with frequent reports of break-through pain
over the past 24 hours 21.Persons at risk are the greater target population for
C. A patient scheduled for exploratory laparotomy this cancer screening programs. Which asymptomatic
morning patient(s) needs extra encouragement to participate in
D. A patient with anticipatory nausea and vomiting for cancer screening? (Choose all that apply).
the past 24 hours A. A 19-year-old white-American female who is sexually
___A__, __C__, __B__, __D__ inactive for a Pap smear
B. A 35-year-old white-American female for an annual
14.In monitoring patients who are at risk for spinal cord mammogram
compression related to tumor growth, what is the most C. A 45-year-old African-American male for an annual
likely early manifestation? prostate-specific antigen
A. Sudden-onset back pain D. A 49year-old African-American male for an annual
B. Motor loss fecal occult blood test
C. Constipation
D. Urinary hesitancy 22.A patient with lung cancer develops syndrome of
inappropriate antidiuretic hormone secretion (SIADH).
15.Chemotherapeutic treatment of acute leukemia is done After reporting symptoms of weight gain, weakness, and
in four phases. Place these phases in the correct order. nausea and vomiting to the physician, you would
A. Maintenance anticipate which initial order for the treatment of this
B. Induction patient?
C. Intensification A. A fluid bolus as ordered
D. Consolidation B. Fluid restrictions as ordered
___B__, __C___, __D___, __A___ C. Urinalysis as ordered
D. Sodium-restricted diet as ordered
16.Which set of classification values indicates the most
extensive and progressed cancer? 23.In caring for a patient with neutropenia, what tasks can
A. T1 N0 M0 be delegated to the nursing assistant? (Choose all that
B. Tis N0 M0 apply).
C. T1 N1 M0 A. Take vital signs every 4 hours
D. T4 N3 M1 B. Report temperature elevation >100.4o F
C. Assess for sore throat, cough, or burning with
17.For a patient with osteogenic sarcoma, you would be urination.
particularly vigilant for elevations in which laboratory D. Gather the supplies to prepare the room for
value? protective isolation.
A. Sodium E. Report superinfections, such as candidiasis
B. Calcium F. Practice good handwashing technique.
C. Potassium
D. Hematocrit 24.A primary nursing responsibility is the prevention of
lung cancer by assisting patients in smoking/tobacco
18.Which of the following cancer patients could potentially cessation. Which tasks would be appropriate to delegate
be placed together as roommates? to the LPN/LVN?
A. A patient with a neutrophil count of 1000/mm3 A. Develop a “quit plan”
B. A patient who underwent debulking of a tumor to B. Explain the application of a nicotine patch
relieve pressure C. Discuss strategies to avoid relapse
C. A patient receiving high-dose chemotherapy after a D. Suggest ways to deal with urges for a tobacco
bone marrow harvest
D. A patient who is post-op laminectomy for spinal cord
compression
1) Nurse April is teaching a client who suspects that she 7) A male client with a cerebellar brain tumor is admitted
has a lump in her breast. The nurse instructs the client to an acute care facility. The nurse formulates a nursing
that a diagnosis of breast cancer is confirmed by: diagnosis of Risk for injury. Which “related-to” phrase
A. breast self-examination. should the nurse add to complete the nursing diagnosis
B. mammography statement?
C. fine needle aspiration. A. Related to visual field deficits
D. chest X-ray. B. Related to difficulty swallowing
C. Related to impaired balance
2) A male client has an abnormal result on a D. Related to psychomotor seizures
Papanicolaou test. After admitting, he read his chart while
the nurse was out of the room, the client asks what 8) Mina, who is suspected of an ovarian tumor is
dysplasia means. Which definition should the nurse scheduled for a pelvic ultrasound. The nurse provides
provide? which preprocedure instruction to the client?
A. Presence of completely undifferentiated tumor cells A. Eat a light breakfast only
that don’t resemble cells of the tissues of their origin B. Maintain an NPO status before the procedure
B. Increase in the number of normal cells in a normal C. Wear comfortable clothing and shoes for the
arrangement in a tissue or an organ procedure
C. Replacement of one type of fully differentiated cell by D. Drink six to eight glasses of water without voiding
another in tissues where the second type normally before the test
isn’t found
D. Alteration in the size, shape, and organization of 9) A male client with a nagging cough makes an
differentiated cells appointment to see the physician after reading that this
symptom is one of the seven warning signs of cancer.
3) Nurse Lucia is providing breast cancer education at a What is another warning sign of cancer?
community facility. The American Cancer Society A. Persistent nausea
recommends that women get mammograms: B. Rash
A. yearly after age 40. C. Indigestion
B. after the birth of the first child and every 2 years D. Chronic ache or pain
thereafter.
C. after the first menstrual period and annually 10) The nurse is instructing the 35 year old client to
thereafter perform a testicular self-examination. The nurse tells the
D. every 3 years between ages 20 and 40 and annually client:
thereafter. A. To examine the testicles while lying down
B. That the best time for the examination is after a
4) A female client is receiving chemotherapy to treat shower
breast cancer. Which assessment finding indicates a fluid C. To gently feel the testicle with one finger to feel for a
and electrolyte imbalance induced by chemotherapy? growth
A. Urine output of 400 ml in 8 hours D. That testicular self-examination should be done at
B. Serum potassium level of 3.6 mEq/L least every 6 months
C. Blood pressure of 120/64 to 130/72 mm Hg
D. Dry oral mucous membranes and cracked lips 11) Nurse Kent is teaching a male client to perform
monthly testicular self-examinations. Which of the
5) A 35 years old client has been receiving chemotherapy following points would be appropriate to make?
to treat cancer. Which assessment finding suggests that A. Testicular cancer is a highly curable type of cancer
the client has developed stomatitis (inflammation of the B. Testicular cancer is very difficult to diagnose.
mouth)? C. Testicular cancer is the number one cause of cancer
A. White, cottage cheese–like patches on the tongue deaths in males.
B. Yellow tooth discoloration D. Testicular cancer is more common in older men.
C. Red, open sores on the oral mucosa
D. Rust-colored sputum 12) A male client undergoes a laryngectomy to treat
laryngeal cancer. When teaching the client how to care for
6) What should a male client over age 52 do to help the neck stoma, the nurse should include which
ensure early identification of prostate cancer? instruction?
A. Have a digital rectal examination and prostate- A. “Keep the stoma uncovered.”
specific antigen (PSA) test done yearly. B. “Keep the stoma dry.”
B. Have a transrectal ultrasound every 5 years. C. “Have a family member perform stoma care initially
C. Perform monthly testicular self-examinations, until you get used to the procedure.”
especially after age 50. D. “Keep the stoma moist.”
D. Have a complete blood count (CBC) and blood urea
nitrogen (BUN) and creatinine levels checked yearly.
13) Rhea, has malignant lymphoma. As part of her D. Urine specific gravity
chemotherapy, the physician prescribes chlorambucil 20) For a female client with newly diagnosed cancer, the
(Leukeran), 10 mg by mouth daily. When caring for the nurse formulates a nursing diagnosis of Anxiety related to
client, the nurse teaches her about adverse reactions to the threat of death secondary to cancer diagnosis. Which
chlorambucil, such as alopecia. How soon after the first expected outcome would be appropriate for this client?
administration of chlorambucil might this reaction occur? A. “Client verbalizes feelings of anxiety.”
A. Immediately B. “Client doesn’t guess at prognosis.”
B. 1 week C. “Client uses any effective method to reduce tension.”
C. 2 to 3 weeks D. “Client stops seeking information.”
D. 1 month
21) The home health care nurse is caring for a male client
14) A female client with cancer is scheduled for radiation with cancer and the client is complaining of acute pain.
therapy. The nurse knows that radiation at any treatment The appropriate nursing assessment of the client’s pain
site may cause a certain adverse effect. Therefore, the would include which of the following?
nurse should prepare the client to expect: A. The client’s pain rating
A. hair loss. B. Nonverbal cues from the client
B. stomatitis C. The nurse’s impression of the client’s pain
C. fatigue D. Pain relief after appropriate nursing intervention
D. vomiting
22) A female client with cancer is receiving chemotherapy
15) Nurse Farah is caring for a client following a and develops thrombocytopenia. The nurse identifies
mastectomy. Which assessment finding indicates that the which intervention as the highest priority in the nursing
client is experiencing a complication related to the plan of care?
surgery? A. Monitoring temperature
A. Pain at the incisional site B. Ambulation three times daily
B. Arm edema on the operative side C. Monitoring the platelet count
C. Sanguineous drainage in the Jackson-Pratt drain D. Monitoring for pathological fractures
D. Complaints of decreased sensation near the
operative site 23) A male client is in isolation after receiving an internal
radioactive implant to treat cancer. Two hours later, the
16) A cervical radiation implant is placed in the client for nurse discovers the implant in the bed linens. What
treatment of cervical cancer. The nurse initiates what should the nurse do first?
most appropriate activity order for this client? A. Stand as far away from the implant as possible and
A. Bed rest call for help.
B. Out of bed ad lib B. Pick up the implant with long-handled forceps and
C. Out of bed in a chair only place it in a lead-lined container.
D. Ambulation to the bathroom only C. Leave the room and notify the radiation therapy
department immediately.
17) Nurse Amy is speaking to a group of women about D. Put the implant back in place, using forceps and a
early detection of breast cancer. The average age of the shield for self-protection, and call for help.
women in the group is 47. Following the American Cancer
Society guidelines, the nurse should recommend that the 24) A 35 years old client with ovarian cancer is prescribed
women: hydroxyurea (Hydrea), an antimetabolite drug.
A. perform breast self-examination annually. Antimetabolites are a diverse group of antineoplastic
B. have a mammogram annually. agents that interfere with various metabolic actions of the
C. have a hormonal receptor assay annually. cell. The mechanism of action of antimetabolites interferes
D. have a physician conduct a clinical examination with:
every 2 years. A. cell division or mitosis during the M phase of the cell
cycle.
18) Nurse Kate is reviewing the complications of B. normal cellular processes during the S phase of the
colonization with a client who has microinvasive cervical cell cycle.
cancer. Which complication, if identified by the client, C. the chemical structure of deoxyribonucleic acid
indicates a need for further teaching? (DNA) and chemical binding between DNA
A. Infection molecules (cell cycle–nonspecific)
B. Hemorrhage D. one or more stages of ribonucleic acid (RNA)
C. Cervical stenosis synthesis, DNA synthesis, or both (cell cycle–
D. Ovarian perforation nonspecific).

19) Nurse Mickey is caring for a client who is 25) For a female client newly diagnosed with radiation-
postoperative following a pelvic exenteration and the induced thrombocytopenia, the nurse should include
physician changes the client’s diet from NPO status to which intervention in the plan of care?
clear liquids. The nurse makes which priority assessment A. Administering aspirin if the temperature exceeds
before administering the diet? 102° F (38.8° C)
A. Bowel sounds B. Inspecting the skin for petechiae once every shift
B. Ability to ambulate C. Providing for frequent rest periods
C. Incision appearance D. Placing the client in strict isolation
26) Nurse Meredith is instructing a premenopausal 33) A female client is undergoing tests for multiple
woman about breast self-examination. The nurse should myeloma. Diagnostic study findings in multiple myeloma
tell the client to do her self-examination: include:
A. at the end of her menstrual cycle. A. a decreased serum creatinine level
B. on the same day each month. B. hypocalcemia
C. on the 1st day of the menstrual cycle. C. Bence Jones protein in the urine.
D. immediately after her menstrual period. D. a low serum protein level.

27) he nurse is admitting a male client with laryngeal 34) Nurse Bea is reviewing the laboratory results of a
cancer to the nursing unit. The nurse assesses for which client diagnosed with multiple myeloma. Which of the
most common risk factor for this type of cancer? following would the nurse expect to note specifically in this
A. Alcohol abuse disorder?
B. Cigarette smoking A. Increased calcium
C. Use of chewing tobacco B. Increased white blood cells
D. Exposure to air pollutants C. Decreased blood urea nitrogen level
D. Decreased number of plasma cells in the bone
28) A male client complains of sporadic epigastric pain, marrow
yellow skin, nausea, vomiting, weight loss, and fatigue.
Suspecting gallbladder disease, the physician orders a 35) Jeovina, with advanced breast cancer is prescribed
diagnostic workup, which reveals gallbladder cancer. tamoxifen (Nolvadex). When teaching the client about this
Which nursing diagnosis may be appropriate for this drug, the nurse should emphasize the importance of
client? reporting which adverse reaction immediately?
A. Anticipatory grieving A. Vision changes
B. Impaired swallowing B. Hearing loss
C. Disturbed body image C. Headache
D. Chronic low self-esteem D. Anorexia

29) Nurse Brian is developing a plan of care for marrow 36) A 34-year-old female client is requesting information
suppression, the major dose-limiting adverse reaction to about mammograms and breast cancer. She isn’t
floxuridine (FUDR). How long after drug administration considered at high risk for breast cancer. What should the
does bone marrow suppression become noticeable? nurse tell this client?
A. 24 hours A. She should have had a baseline mammogram before
B. 2 to 4 days age 30.
C. 7 to 14 days B. She should eat a low-fat diet to further decrease her
D. 21 to 28 days risk of breast cancer.
C. She should perform breast self-examination during
30) The nurse is interviewing a male client about his past the first 5 days of each menstrual cycle.
medical history. Which preexisting condition may lead the D. When she begins having yearly mammograms,
nurse to suspect that a client has colorectal cancer? breast self-examinations will no longer be necessary.
A. Duodenal ulcers
B. Hemorrhoids 37) During chemotherapy, an oncology client has a
C. Weight gain nursing diagnosis of impaired oral mucous membrane
D. Polyps related to decreased nutrition and immunosuppression
secondary to the cytotoxic effects of chemotherapy.
31) The ABCD method offers one way to assess skin Which nursing intervention is most likely to decrease the
lesions for possible skin cancer. What does the A stand pain of stomatitis?
for? A. Recommending that the client discontinue
A. Actinic chemotherapy
B. Asymmetry B. Providing a solution of hydrogen peroxide and water
C. Arcus for use as a mouth rinse
D. Assessment C. Monitoring the client’s platelet and leukocyte counts
D. Checking regularly for signs and symptoms of
32) A client, age 41, visits the gynecologist. After stomatitis
examining her, the physician suspects cervical cancer.
The nurse reviews the client’s history for risk factors for 38) When caring for a male client diagnosed with a brain
this disease. Which history finding is a risk factor for tumor of the parietal lobe, the nurse expects to assess:
cervical cancer? A. short-term memory impairment.
A. Onset of sporadic sexual activity at age 17 B. tactile agnosia.
B. Spontaneous abortion at age 19 C. seizures
C. Pregnancy complicated with eclampsia at age 27 D. contralateral homonymous hemianopia.
D. Human papillomavirus infection at age 32
39) Vanessa, a community health nurse conducts a health 45) A female client diagnosed with multiple myeloma and
promotion program regarding testicular cancer to the client asks the nurse about the diagnosis. The nurse
community members. The nurse determines that further bases the response on which description of this disorder?
information needs to be provided if a community member A. Altered red blood cell production
states that which of the following is a sign of testicular B. Altered production of lymph nodes
cancer? C. Malignant exacerbation in the number of leukocytes
A. Alopecia D. Malignant proliferation of plasma cells within the
B. Back pain bone
C. Painless testicular swelling
D. Heavy sensation in the scrotum 46) A male client is diagnosed as having a bowel tumor
and several diagnostic tests are prescribed. The nurse
40) During the admission assessment of a 35 year old understands that which test will confirm the diagnosis of
client with advanced ovarian cancer, the nurse recognizes malignancy?
which symptom as typical of the disease? A. Biopsy of the tumor
A. Diarrhea B. Abdominal ultrasound
B. Hypermenorrhea C. Magnetic resonance imaging
C. Abdominal bleeding D. Computerized tomography scan
D. Abdominal distention
47) The male client is receiving external radiation to the
41) Nurse April is teaching a group of women to perform neck for cancer of the larynx. The most likely side effect to
breast self-examination. The nurse should explain that the be expected is:
purpose of performing the examination is to discover: A. Dyspnea
A. cancerous lumps. B. Diarrhea
B. areas of thickness or fullness. C. Sore throat
C. changes from previous self-examinations. D. Constipation
D. fibrocystic masses.
48) Mr. Miller has been diagnosed with bone cancer. You
42) A male client is receiving the cell cycle–nonspecific know this type of cancer is classified as:
alkylating agent thiotepa (Thioplex), 60 mg weekly for 4 A. sarcoma
weeks by bladder instillation as part of a B. lymphoma
chemotherapeutic regimen to treat bladder cancer. The C. carcinoma
client asks the nurse how the drug works. How does D. melanoma
thiotepa exert its therapeutic effects?
A. It interferes with deoxyribonucleic acid (DNA) 49) Nina, an oncology nurse educator is speaking to a
replication only. women’s group about breast cancer. Questions and
B. It interferes with ribonucleic acid (RNA) transcription comments from the audience reveal a misunderstanding
only. of some aspects of the disease. Various members of the
C. It interferes with DNA replication and RNA audience have made all of the following statements.
transcription. Which one is accurate?
D. It destroys the cell membrane, causing lysis. A. Mammography is the most reliable method for
detecting breast cancer.
43) A female client is hospitalized for insertion of an B. Breast cancer is the leading killer of women of
internal cervical radiation implant. While giving care, the childbearing age.
nurse finds the radiation implant in the bed. The initial C. Breast cancer requires a mastectomy.
action by the nurse is to: D. Men can develop breast cancer.
A. Call the physician
B. Reinsert the implant into the vagina immediately 50) Gian, a community health nurse is instructing a group
C. Pick up the implant with gloved hands and flush it of female clients about breast self-examination. The nurse
down the toilet instructs the client to perform the examination:
D. Pick up the implant with long-handled forceps and A. At the onset of menstruation
place it in a lead container. B. Every month during ovulation
C. Weekly at the same time of day
44) A male client is admitted to the hospital with a D. 1 week after menstruation begins
suspected diagnosis of Hodgkin’s disease. Which
assessment findings would the nurse expect to note 51) The nurse is preparing for a female client for magnetic
specifically in the client? resonance imaging (MRI) to confirm or rule out a spinal
A. Fatigue cord lesion. During the MRI scan, which of the following
B. Weakness would pose a threat to the client?
C. Weight gain A. The client lies still.
D. Enlarged lymph nodes B. The client asks questions.
C. The client hears thumping sounds.
D. The client wears a watch and wedding band.
52) A female client is receiving methotrexate (Mexate), 12 59) The nurse is caring for a female client experiencing
g/m2 I.V., to treat osteogenic carcinoma. During neutropenia as a result of chemotherapy and develops a
methotrexate therapy, the nurse expects the client to plan of care for the client. The nurse plans to:
receive which other drug to protect normal cells? A. Restrict all visitors
A. probenecid (Benemid) B. Restrict fluid intake
B. cytarabine (ara-C, cytosine arabinoside [Cytosar-U] C. Teach the client and family about the need for hand
C. thioguanine (6-thioguanine, 6-TG) hygiene
D. leucovorin (citrovorum factor or folinic acid D. Insert an indwelling urinary catheter to prevent skin
[Wellcovorin]) breakdown

53) The female client who has been receiving radiation 60) Nurse Joy is caring for a client with an internal
therapy for bladder cancer tells the nurse that it feels as if radiation implant. When caring for the client, the nurse
she is voiding through the vagina. The nurse interprets should observe which of the following principles?
that the client may be experiencing: A. Limit the time with the client to 1 hour per shift
A. Rupture of the bladder B. Do not allow pregnant women into the client’s room
B. The development of a vesicovaginal fistula C. Remove the dosimeter badge when entering the
C. Extreme stress caused by the diagnosis of cancer client’s room
D. Altered perineal sensation as a side effect of D. Individuals younger than 16 years old may be
radiation therapy allowed to go in the room as long as they are 6 feet
away from the client
54) A female client with cancer is being evaluated for
possible metastasis. Which of the following is one of the 1) Can a mammography detect breast cancer before you
most common metastasis sites for cancer cells? can even see it.
A. Liver A. Yes
B. Colon B. No
C. Reproductive tract C. Sometimes
D. White blood cells (WBCs) D. What is a mammography?

55) Nurse Cecilia is caring for a client who has undergone 2) Jeovina, with advanced breast cancer is prescribed
a vaginal hysterectomy. The nurse avoids which of the tamoxifen (Nolvadex). When teaching the client about this
following in the care of this client? drug, the nurse should emphasize the importance of
A. Elevating the knee gatch on the bed reporting which adverse reaction immediately?
B. Assisting with range-of-motion leg exercises A. Vision changes
C. Removal of antiembolism stockings twice daily B. Hearing loss
D. Checking placement of pneumatic compression C. Headache
boots D. Anorexia

56) Which intervention is appropriate for the nurse caring 3) Noninvasive Breast Cancer is when..
for a male client in severe pain receiving a continuous I.V. A. the cancer cells have spread to other parts of your
infusion of morphine? body
A. Assisting with a naloxone challenge test before B. the cancer cells have stayed from their origin and
therapy begins have not spread
B. Discontinuing the drug immediately if signs of
dependence appear 4) Can family history of breast cancer increase the
C. Changing the administration route to P.O. if the client chances of the family members?
can tolerate fluids A. No
D. Obtaining baseline vital signs before administering B. Yes
the first dose C. Only from the mother’s side
D. Only from the dad’s side
57) Sarah, a hospice nurse visits a client dying of ovarian
cancer. During the visit, the client expresses that “If I can 5) The nurse is teaching the client about breast self-
just live long enough to attend my daughter’s graduation, examination. Which observation should the client be
I’ll be ready to die.” Which phrase of coping is this client taught to recognize when doing the examination for
experiencing? detection of breast cancer?
A. Anger A. tender, movable lump
B. Denial B. pain on breast self-examination
C. Bargaining C. round, well-defined lump
D. Depression D. dimpling of the breast tissue

58) The client with leukemia is receiving busulfan 6) Which of the following terms is used to describe
(Myleran) and allopurinol (Zyloprim). The nurse tells the removal of the breast tissue and an axillary lymph node
client that the purpose if the allopurinol is to prevent: dissection leaving muscular structure intact as surgical
A. Nausea treatment of breast cancer?
B. Alopecia A. Modified radical mastectomy
C. Vomiting B. Segmental mastectomy
D. Hyperuricemia C. Total mastectomy
D. Radical mastectomy C. Underarm
7) Nurse Meredith is instructing a premenopausal woman D. Leg
about breast self-examination. The nurse should tell the 15) Which of the following are used in diagnosing breast
client to do her self-examination: cancer?
A. at the end of her menstrual cycle. A. Mammogram
B. on the same day each month. B. Ultrasound
C. on the 1st day of the menstrual cycle. C. MRI
D. immediately after her menstrual period. D. Biopsy
E. All of the above
8) Hormonal agents are used to treat some cancers. An
example would be: 16) A female client is receiving chemotherapy to treat
A. Thyroxine to treat thyroid cancer. breast cancer. Which assessment finding indicates a fluid
B. ACTH to treat adrenal carcinoma. and electrolyte imbalance induced by chemotherapy?
C. Estrogen antagonists to treat breast cancer. A. Urine output of 400 ml in 8 hours
D. Glucagon to treat pancreatic carcinoma. B. Serum potassium level of 3.6 mEq/L
C. Blood pressure of 120/64 to 130/72 mm Hg
9) What does regional refer to in relation to breast D. Dry oral mucous membranes and cracked lips
cancer?
A. Metastases 17) Maria refuses to acknowledge that her breast was
B. Lymph nodes removed. She believes that her breast is intact under the
C. A tumor within the breast dressing. The nurse should
D. A tumor is within both breasts A. call the MD to change the dressing so Kathy can see
the incision
10) A client with breast cancer is returned to the room B. recognize that Kathy is experiencing denial, a normal
following a right total mastectomy. The nurse should: stage of the grieving process
A. Elevate the client’s right arm on pillows C. reinforce Kathy’s belief for several days until her
B. Place the client’s right arm in a dependent sling body can adjust to stress of surgery.
C. Keep the client’s right arm on the bed beside her D. remind Kathy that she needs to accept her diagnosis
D. Place the client’s right arm across her body so that she can begin rehabilitation exercises.

11) The community health nurse implemented a new 18) Nina, an oncology nurse educator is speaking to a
program about effective breast cancer screening women’s group about breast cancer. Questions and
technique for the female personnel of the health comments from the audience reveal a misunderstanding
department of Valenzuela. Which of the following of some aspects of the disease. Various members of the
technique should the nurse consider to be of the lowest audience have made all of the following statements.
priority? Which one is accurate?
A. Yearly breast exam by a trained professional A. Mammography is the most reliable method for
B. Detailed health history to identify women at risk detecting breast cancer.
C. Screening mammogram every year for women over B. Breast cancer is the leading killer of women of
age 50 childbearing age.
D. Screening mammogram every 1-2 years for women C. Breast cancer requires a mastectomy.
over age of 40. D. Men can develop breast cancer.

12) The nurse is speaking to a group of women about 19) The community health nurse implemented a new
early detection of breast cancer. The average age of the program about effective breast cancer screening
women in the group is 47. Following the American Cancer technique for the female personnel of the health
Society (ACS) guidelines, the nurse should recommend department of Valenzuela. Which of the following
that the women: technique should the nurse consider to be of the lowest
A. perform breast self-examination annually priority?
B. have a mammogram annually A. Yearly breast exam by a trained professional
C. have a hormonal receptor assay annually B. Detailed health history to identify women at risk
D. have a physician conduct a clinical evaluation every C. Screening mammogram every year for women over
2 years age 50
D. Screening mammogram every 1-2 years for women
13) What is the purpose of using hormonal therapy for over age of 40
patients with breast cancer cells?
A. To replace the hormones and add testosterone. 20) The 2000 NIH Consensus Development Conference
B. To produce more hormones by adding testosterone Statement states that what percentage of women with
along with estrogen and progesterone. invasive breast cancer should consider the option of
C. To block the supply of estrogen or pregesterone to systemic chemotherapy, not just women whose tumors
the cells. are greater than 1cm in size?
A. 100% (all)
14) Breast Cancer is most likely to spread to your B. 75%
____________area. C. 50%
A. Face D. 25%
B. Stomach
B. Lung cancer
C. Brain cancer
21) The statistics decrease for getting breast cancer as D. Colon and rectal cancer
one gets older in age. 31) Is early detection of breast cancer your best chance of
A. True survival?
B. False A. No
B. Yes
22) Can men get breast cancer? C. Sometimes
A. Yes, always D. I Don’t Know
B. No, never
C. Yes, but not likely 32) A 34-year-old female client is requesting information
D. I don’t know about mammograms and breast cancer. She isn’t
considered at high risk for breast cancer. What should the
23) What does local breast cancer refer to? nurse tell this client?
A. Metastases A. She should have had a baseline mammogram before
B. The chest wall age 30.
C. The breast B. She should eat a low-fat diet to further decrease her
D. Lymph nodes risk of breast cancer.
C. She should perform breast self-examination during
24) What is the percentage of breast tissue that can be the first 5 days of each menstrual cycle.
removed and rearranged to have the breast still looking D. When she begins having yearly mammograms,
like a breast? breast self-examinations will no longer be necessary.
A. 60%
B. 15% 33) Nurse Amy is speaking to a group of women about
C. 30% early detection of breast cancer. The average age of the
D. 45% women in the group is 47. Following the American Cancer
Society guidelines, the nurse should recommend that the
25) Breast reconstruction is done women:
after_____________surgery. A. perform breast self-examination annually.
A. Lumpectomy B. have a mammogram annually.
B. Mastectomy C. have a hormonal receptor assay annually.
D. have a physician conduct a clinical examination
26) Breast Cancer is the most common reason for death every 2 years.
in what ages?
A. 0-4 34) The nurse recognizes which of the following
B. 85-99 statements as accurately reflecting a risk factor for breast
C. 55-84 cancer?
D. 15-54 A. Mother affected by cancer before 60 years of age
B. Onset of menses before 14 years of age
27) Ductal lavage is used for C. Multiparity
A. women at higher risk for benign proliferative breast D. No alcohol consumption
disease.
B. women at low risk for breast cancer. 35) Only women can get Breast Cancer.
C. screening women over age 65. A. True
D. women with breast implants. B. False

28) Maria Sison, 40 years old, single, was admitted to the 36) What is Breast Cancer?
hospital with a diagnosis of Breast Cancer. She was A. A tumor that developes in the breast area
scheduled for radical mastectomy. Nursing care during B. A very common disease among women
the preoperative period should consist of C. A deadly disease
A. assuring Maria that she will be cured of cancer D. All of the above
B. assessing Maria’s expectations and doubts
C. maintaining a cheerful and optimistic environment 37) A male client with a nagging cough makes an
D. keeping Maria’s visitors to a minimum so she can appointment to see the physician after reading that this
have time for herself symptom is one of the seven warning signs of cancer.
What is another warning sign of cancer?
29) What is a symptom of Breast Cancer? A. Persistent nausea
A. Itchy throat B. Rash
B. Lump in underarm or breast area. C. Indigestion
C. Pain in the arm D. Chronic ache or pain

30) A 37-year-old client with uterine cancer asks the 38)A client receiving chemotherapy for breast cancer has
nurse, “Which is the most common type of cancer in an order for Zofran (ondansetron) 8mg PO to be given 30
women?” The nurse replies that it’s breast cancer. Which minutes before induction of the chemotherapy. The
type of cancer causes the most deaths in women? purpose of the medication is to:
A. Breast cancer A. Prevent anemia
B. Promote relaxation A. Adenocarcinoma
C. Prevent nausea B. Large cell carcinoma
D. Increase neutrophil counts C. Squamous cell carcinoma
39) During a breast examination, which finding most D. Small cell carcinoma
strongly suggests that the Luz has breast cancer? 2) A 37-year-old client with uterine cancer asks the nurse,
A. Slight asymmetry of the breasts. “Which is the most common type of cancer in women?”
B. A fixed nodular mass with dimpling of the overlying The nurse replies that it’s breast cancer. Which type of
skin cancer causes the most deaths in women?
C. Bloody discharge from the nipple A. Breast cancer
D. Multiple firm, round, freely movable masses that B. Lung cancer
change with the menstrual cycle C. Brain cancer
D. Colon and rectal cancer
40) 1 in ____ women get Breast Cancer.
A. 7 3) A 64-year-old woman returns to your clinic for
B. 2 continued evaluation of a chronic cough, which she has
C. 8 been experiencing for the past 2 months. A chest x-ray
D. 4 obtained yesterday shows a spiculated right middle lobe
mass with mediastinal lymphadenopathy. In addition,
41) Nurse Lucia is providing breast cancer education at a today the patient reports right leg pain of new onset. You
community facility. The American Cancer Society are concerned about metastatic disease.
recommends that women get mammograms: For this patient, which of the following approaches to
A. yearly after age 40. diagnostic imaging would be most appropriate for the
B. after the birth of the first child and every 2 years evaluation of possible metastatic disease?
thereafter. A. CT scan of the chest
C. after the first menstrual period and annually B. CT scan of the chest, head, and adrenal glands
thereafter. C. CT scan of the chest, head, and adrenal glands, and
D. every 3 years between ages 20 and 40 and annually a bone scan
thereafter. D. CT scan of the chest and head, and a bone scan

42) A female client was diagnosed with breast cancer. It is 4) Which of the following types of lung cancer is
found to be stage IV, and a modified mastectomy is characterized as fast growing and tending to arise
performed. After the procedure, what behaviors could the peripherally?
nurse expects the client to display? A. Large cell carcinoma
A. Denial of the possibility of carcinoma. B. Bronchioalveolar carcinoma
B. Signs of grief reaction. C. Adenocarcinoma
C. Relief that the operation is over. D. Squamous cell carcinoma
D. . Signs of deep depression.
5) A patient is scheduled for a magnetic resonance
43) A 52 yr-old female tells the nurse that she has found a imaging (MRI) scan for suspected lung cancer. Which of
painless lump in her right breast during her monthly self- the following is a contraindication to the study for this
examination. Which assessment finding would strongly patient?
suggest that this client’s lump is cancerous? A. The patient is allergic to shellfish.
A. eversion of the right nipple and a mobile mass B. The patient has a pacemaker.
B. nonmobile mass with irregular edges C. The patient suffers from claustrophobia.
C. mobile mass that is oft and easily delineated D. The patient takes anti-psychotic medication.
D. nonpalpable right axillary lymph nodes
6) A 67-year-old client who is receiving chemotherapy for
44) Nurse April is teaching a group of women to perform lung cancer is admitted to the hospital with
breast self-examination. The nurse should explain that the thrombocytopenia. While you are taking the admission
purpose of performing the examination is to discover: history, the client makes these statements. Which
A. cancerous lumps. statement is of most concern?
B. areas of thickness or fullness. A. “I’ve noticed that I bruise more easily since the
C. changes from previous self-examinations. chemotherapy started.”
D. fibrocystic masses. B. “My bowel movements are soft and dark brown in
color.”
45) Nurse April is teaching a client who suspects that she C. “I take one aspirin every morning because of my
has a lump in her breast. The nurse instructs the client history of angina.”
that a diagnosis of breast cancer is confirmed by: D. “My appetite has decreased since the chemotherapy
A. breast self-examination. strated.”
B. mammography.
C. fine needle aspiration. 7) What is true about interventional pulmonology?
D. chest X-ray. A. Looks at reasons for shortness of breath. This is not
only used for lung cancer; it can be used for breast
1) Which of the following types of lung cancer is the most cancer, colon cancer, renal cancer and melanoma.
prevalent carcinoma of the lung for both men and B. This increases the likelihood for cure by 29%.
women?
C. CTCA was the first to combine interventional A. Helping the client deal with depression secondary to
pulmonology with various treatments. the diagnosis and its treatment
B. Explaining that the reactions to chemotherapy are
minimal
C. Careful observation of the IV site of the
8) Gina, a home health nurse is visiting a home care client administration of the drugs
with advanced lung cancer. Upon assessing the client, the D. Careful attention to blood count results
nurse discovers wheezing, bradycardia, and a respiratory 17) A 56-year-old man is worried about his risk of
rate of 10 breaths/minute. These signs are associated developing lung cancer. He has no medical problems, but
with which condition? he does report having smoked one pack of cigarettes a
A. Hypoxia day for 4 years while in college. He stopped smoking
B. Delirium more than 30 years ago.
C. Hyperventilation Of the following which is the most accurate statement
D. Semiconsciousness regarding this patient’s risk of developing lung cancer?
A. It is equal to someone who has never smoked
9) What percentage of lung cancer is caused by smoking? B. In any age group, the risk of developing lung cancer
A. 50% declines after stopping smoking
B. 63% C. The annual lung cancer mortality of smokers is 10
C. 87% times that of nonsmokers
D. 94% D. His risk of developing lung cancer is only slightly
greater than that of a nonsmoker
10) What is the benefit of a navigational bronchoscopy?
A. Performs biopsies 18) Lung cancer is the top killer of cancer.
B. Delivers High Dose Radiation (HDR) A. True
C. Explains the distance that the bronchoscope can B. False
travel into the lung.
D. All of the above 19) A client with lung cancer has received oxycodone 10
11) Of the four basic cell types of lung cancer listed mg orally for pain. When the student nurse assesses the
below, which is always associated with smoking? client, which finding should you instruct the student to
A. adenocarcinoma report immediately?
B. squamous cell carcinoma (epidermoid) A. Respiratory rate of 8 to 10 per minute
C. undifferenciated carcinoma B. Pain level decreased from 6/10 to 2/10
D. bronchoalveolar carcinoma C. Client requests room door be closed.
D. Heart rate 90-100 per minute
12) In terms of lung cancer, what is intratumoral
chemotherapy used for? 20) A 62-year-old woman presents to your clinic for a
A. Small Cell Lung Cancer regularly scheduled examination. She has no new
B. Non-Small Cell Lung Cancer complaints, but she is concerned about her risk of
developing lung cancer. She has smoked one pack of
13) When a patient in the terminal stages of lung cancer cigarettes a day since she was 18 years old. She asks
begins to exhibit loss of consciousness, a major nursing whether she should be given any tests to screen for lung
priority is to: cancer.
A. Protect the patient from injury For this patient, which of the following statements is true?
B. Insert an airway A. Chest x-rays are a cost-effective and accurate way
C. Elevate the head of the bed to screen for lung cancer
D. Withdraw all pain medications B. Spiral computed tomography scanning is currently
recommended for screening in asymptomatic
14) What is an autofluorescence bronchoscopy? patients
A. This shows where treatment is being delivered to C. Benign nodules are rarely seen on spiral CT, making
within the lung. it ideal for cancer screening
B. This lights up cancer cells and it can show cells that D. At present, there are no recommended radiographic
haven’t grown into a tumor yet. studies to screen for lung cancer
C. This lights up the path that a bronchoscope can
follow. 21) A client with lung cancer is admitted in the nursing
care unit. The husband wants to know the condition of his
15) Which of the following is the most common type of wife. How should the nurse respond to the husband?
lung cancer? A. Find out what information he already has.
A. Large cell B. Suggest that he discuss it with his wife.
B. Adenocarcinoma C. Refer him to the doctor.
C. Oat cell D. Refer him to the nurse in charge.
D. Squamous cell

16) Chemotherapy may be used in combination with 22) The nurse is conducting an education session for a
surgery in the treatment of lung cancer. Special nursing group of smokers in a “stop smoking” class. Which finding
considerations with chemotherapy include all but which of would the nurse state as a common symptom of lung
the following? cancer? :
A. Dyspnea on exertion
B. Foamy, blood-tinged sputum  Have a transrectal ultrasound every 5 years
C. Wheezing sound on inspiration  Perform monthly testicular self-examinations,
D. Cough or change in a chronic cough especially after age 50
 Have a digital rectal examination and prostate-
specific antigen (PSA) test done yearly
 Have a CBC and BUN and creatinine levels
checked yearly
23) Which of the following symptoms is most The ABCD method offers one way to assess skin lesions
characteristic of a client with a cancer of the lung? for possible skin cancer. What does the A stand for?
A. air hunger  Assessment
B. exertional dyspnea  Arcus
C. cough with night sweats
 Actinic
D. persistent changing cough
 Asymmetry
24) What does an Endobronchial Ultrasound help do and
Mina, who is suspected of an ovarian tumor is scheduled
how does it work?
for a pelvic ultrasound. The nurse provides which pre-
A. An Endobronchial Ultrasound helps with the staging
procedure instruction to the client?
of a tumor. A bronchoscope is inserted into the lung
with an ultrasound on the tip. There is a slot that  Wear comfortable clothing and shoes for the
houses a needle to biopsy the lymph nodes. procedure
B. An Endobronchial Ultrasound helps take images of  Maintain an NPO status before the procedure
the lung. A long scope is inserted directly into the  Drink six to eight glasses of water without voiding
chest cavity. The scope takes 30 images with a 360 before the test
degree rotation. The images are sent to a computer  Eat a light breakfast only
where the oncologist is able to examine the images.
C. An Endobronchial Ultrasound helps with the A female client with cancer is being evaluated for possible
placement of fiducial markers. An x-ray is taken of metastasis. Which of the following is one of the most
the lung and lymph nodes, the endobronchial common metastasis sites for cancer cells?
ultrasound is then inserted in the area of the tumor(s)  Colon
for implantation of fiducial markers.  Liver
 Reproductive tract
25) The most common lethal cancer in males between  White blood cells (WBCs)
their fifth and seventh decades is:
A. cancer of the prostate A male client is receiving the cell cycle-nonspecific
B. cancer of the lung alkylating agent Thioplex (thiotepa), 60 mg weekly for 4
C. cancer of the pancreas weeks by bladder installation as part of a
D. cancer of the bowel chemotherapeutic regimen to treat bladder cancer. The
client asks the nurse how the drug works. How does
26) What is the most common reason for bronchoscopes thiotepa exert its therapeutic effects?
not being able to reach a tumor within the lung?  It interferes with deoxyribonucleic acid (DNA)
A. Scar tissue replication only
B. Malformations of the lung  It interferes with ribonucleic acid (RNA)
C. The tumor is too small transcription only
D. Due to small bronchi  It interferes with DNA replication and RNA
transcription
27) Antonio with lung cancer develops Horner’s syndrome  It destroys the cell membranes, causing lysis
when the tumor invades the ribs and affects the
sympathetic nerve ganglia. When assessing for signs and A female client with cancer is scheduled for radiation
symptoms of this syndrome, the nurse should note: therapy. The nurse knows that radiation at any treatment
A. miosis, partial eyelid ptosis, and anhidrosis on the site may cause a certain adverse effect. Therefore, the
affected side of the face. nurse should prepare the client to expect:
B. chest pain, dyspnea, cough, weight loss, and fever.  Fatigue
C. arm and shoulder pain and atrophy of arm and hand  Vomiting
muscles, both on the affected side.
 Hair loss
D. hoarseness and dysphagia.
 Stomatitis
The nurse is interviewing a male client about his past A 56-year-old woman is currently receiving radiation
medical history. Which preexisting condition may lead the therapy to the chest wall for recurrent breast cancer. She
nurse to suspect that a client has colorectal cancer? calls her health care provider to report that she has pain
 Polyps while swallowing and burning and tightness in her chest.
 weight gain Which of the following complications of radiation therapy
 Hemorrhoids is A. Radiation enteritis likely responsible for her
 Duodenal ulcers symptoms?
 Radiation enteritis
What should a male client over age 52 do to help ensure  Stomatitis
early identification of prostate cancer?  Esophagitis
 Hiatal hernia symptom is one of the seven warning signs of cancer.
What is another warning sign of cancer?
When caring for a male client diagnosed with a brain  rash
tumor of the parietal lobe, the nurse expects to assess:  Indigestion
 Seizures  chronic ache or pain
 Tactile agnosia  Persistent nausea
 Short-term memory impairment
 Contralateral homonymous hemianopia
Nurse Joy is caring for a client with an internal radiation
implant. When caring for the client, the nurse should One of the most serious blood coagulation complications
observe which of the following principles? for individuals with cancer and for those undergoing
 Remove the dosimeter badge when entering the cancer treatments is disseminated intravascular
client's room coagulation (DIC). The most common cause of this
 Individual's younger than 16 yr may be allowed to bleeding disorder is:
go in the room as long as they are 6 feet away  Brain metastasis
from the client  Sepsis
 Limit the time with the client to 1 hour per shift  Intravenous heparin therapy
 Do not allow pregnant women into the client's  underlying liver disease
room
The male client is receiving external radiation to the neck
For a female client with newly diagnosed cancer, the for cancer of the larynx. The most likely side effect to be
nurse formulates a nursing diagnosis of anxiety related to expected is:
the threat of death secondary to a cancer diagnosis.  Diarrhea
Which expected outcome would be appropriate for this  dyspnea
client?  Constipation
 "Client stops seeking information."  Sore throat
 "Client uses any effective method to reduce
tension." Nurse Kent is teaching a male client to perform monthly
 "Client doesn't guess at prognosis." testicular self-examinations. Which of the following points
 "Client verbalizes feeling of anxiety." would be appropriate to make?
 Testicular cancer is a highly curable type of cancer
Nurse Lucia is providing breast cancer education at a  Testicular cancer is very difficult to diagnose
community facility. The American Cancer Society  Testicular cancer is the number one cause of
recommends that women get mammograms: cancer deaths in males
 After the first menstrual period and annually  Testicular cancer is more common in older men
thereafter
 Yearly after age 40 Nurse April is teaching a group of women to perform
 Every 3 years between ages 20 and 40 and breast self-examination. The nurse should explain that the
annually thereafter purpose of performing the examination is to discover:
 After the birth of the first child and every 2 years  Fibrocystic masses
thereafter  Changes from previous self-examinations
 Areas of thickness or fullness
Nurse April is teaching a client who suspects that she has  Cancerous lumps
a lump in her breast. The nurse instructs the client that a
diagnosis of breast cancer is confirmed by: For a female client newly diagnosed with radiation-
 Breast self-examination induced thrombocytopenia, the nurse should include
 Mammography which intervention in the plan of care?
 Fine needle aspiration  Inspecting the skin for petechiae once every shift
 chest x-ray  Placing the client in strict isolation
 Providing for frequent rest periods
Vanessa, a community health nurse conducts a health  Administering aspirin if the temperature exceeds
promotion program regarding testicular cancer to 102 degrees F (38.8 C)
community members. The nurse determines that further
information needs to be provided if a community member
states that which of the following is a sign of testicular
cancer?
 back pain
 alopecia
 Heavy sensation in the scrotum
 Painless testicular swelling

A male client with a nagging cough makes an


appointment to see the physician after reading that this

You might also like